Determine whether the series $sum_{n=1}^{infty} frac{ln(n)}{n^2 +1}$ converges or not.












2












$begingroup$


Determine whether the series $sum_{n=1}^{infty} frac{ln(n)}{n^2 +1}$ converges or not.



** My trial **
I tried dividing $frac{ln(n)}{n^2 +1}$ by $1/n^2$ and finding the limit which was $infty$ so I could not use the limit comparison test and this idea did not work.



Could anyone give me a hint for studying the convergence of this series?










share|cite|improve this question











$endgroup$












  • $begingroup$
    do you mean $$sum_{n=1}^{infty}frac{ln n}{n^2+1}$$?
    $endgroup$
    – clathratus
    Jan 16 at 4:52










  • $begingroup$
    Hint: Use the inequality $ln n < n$ in the following way $ln n = 2 ln sqrt{n} < 2 sqrt{n}$
    $endgroup$
    – RRL
    Jan 16 at 4:54










  • $begingroup$
    @clathratus yes sorry I corrected it.
    $endgroup$
    – hopefully
    Jan 16 at 4:56


















2












$begingroup$


Determine whether the series $sum_{n=1}^{infty} frac{ln(n)}{n^2 +1}$ converges or not.



** My trial **
I tried dividing $frac{ln(n)}{n^2 +1}$ by $1/n^2$ and finding the limit which was $infty$ so I could not use the limit comparison test and this idea did not work.



Could anyone give me a hint for studying the convergence of this series?










share|cite|improve this question











$endgroup$












  • $begingroup$
    do you mean $$sum_{n=1}^{infty}frac{ln n}{n^2+1}$$?
    $endgroup$
    – clathratus
    Jan 16 at 4:52










  • $begingroup$
    Hint: Use the inequality $ln n < n$ in the following way $ln n = 2 ln sqrt{n} < 2 sqrt{n}$
    $endgroup$
    – RRL
    Jan 16 at 4:54










  • $begingroup$
    @clathratus yes sorry I corrected it.
    $endgroup$
    – hopefully
    Jan 16 at 4:56
















2












2








2





$begingroup$


Determine whether the series $sum_{n=1}^{infty} frac{ln(n)}{n^2 +1}$ converges or not.



** My trial **
I tried dividing $frac{ln(n)}{n^2 +1}$ by $1/n^2$ and finding the limit which was $infty$ so I could not use the limit comparison test and this idea did not work.



Could anyone give me a hint for studying the convergence of this series?










share|cite|improve this question











$endgroup$




Determine whether the series $sum_{n=1}^{infty} frac{ln(n)}{n^2 +1}$ converges or not.



** My trial **
I tried dividing $frac{ln(n)}{n^2 +1}$ by $1/n^2$ and finding the limit which was $infty$ so I could not use the limit comparison test and this idea did not work.



Could anyone give me a hint for studying the convergence of this series?







calculus sequences-and-series






share|cite|improve this question















share|cite|improve this question













share|cite|improve this question




share|cite|improve this question








edited Jan 16 at 5:08









Aaron Quitta

328214




328214










asked Jan 16 at 4:47









hopefullyhopefully

249114




249114












  • $begingroup$
    do you mean $$sum_{n=1}^{infty}frac{ln n}{n^2+1}$$?
    $endgroup$
    – clathratus
    Jan 16 at 4:52










  • $begingroup$
    Hint: Use the inequality $ln n < n$ in the following way $ln n = 2 ln sqrt{n} < 2 sqrt{n}$
    $endgroup$
    – RRL
    Jan 16 at 4:54










  • $begingroup$
    @clathratus yes sorry I corrected it.
    $endgroup$
    – hopefully
    Jan 16 at 4:56




















  • $begingroup$
    do you mean $$sum_{n=1}^{infty}frac{ln n}{n^2+1}$$?
    $endgroup$
    – clathratus
    Jan 16 at 4:52










  • $begingroup$
    Hint: Use the inequality $ln n < n$ in the following way $ln n = 2 ln sqrt{n} < 2 sqrt{n}$
    $endgroup$
    – RRL
    Jan 16 at 4:54










  • $begingroup$
    @clathratus yes sorry I corrected it.
    $endgroup$
    – hopefully
    Jan 16 at 4:56


















$begingroup$
do you mean $$sum_{n=1}^{infty}frac{ln n}{n^2+1}$$?
$endgroup$
– clathratus
Jan 16 at 4:52




$begingroup$
do you mean $$sum_{n=1}^{infty}frac{ln n}{n^2+1}$$?
$endgroup$
– clathratus
Jan 16 at 4:52












$begingroup$
Hint: Use the inequality $ln n < n$ in the following way $ln n = 2 ln sqrt{n} < 2 sqrt{n}$
$endgroup$
– RRL
Jan 16 at 4:54




$begingroup$
Hint: Use the inequality $ln n < n$ in the following way $ln n = 2 ln sqrt{n} < 2 sqrt{n}$
$endgroup$
– RRL
Jan 16 at 4:54












$begingroup$
@clathratus yes sorry I corrected it.
$endgroup$
– hopefully
Jan 16 at 4:56






$begingroup$
@clathratus yes sorry I corrected it.
$endgroup$
– hopefully
Jan 16 at 4:56












1 Answer
1






active

oldest

votes


















2












$begingroup$

Compare with $$sum_{n=1}^{infty}frac{1}{n^{1.5}}$$






share|cite|improve this answer









$endgroup$













  • $begingroup$
    you mean comparison not limit comparison?
    $endgroup$
    – hopefully
    Jan 16 at 4:56










  • $begingroup$
    Either way. This series converges. With direct comparison, the OP series has smaller terms (eventually). With a limit comparison, the ratio of the OP terms to the terms here converges to $0$. Either implies the OP series converges.
    $endgroup$
    – alex.jordan
    Jan 16 at 5:06










  • $begingroup$
    but if I use the direct comparison test how can I compare them?
    $endgroup$
    – hopefully
    Jan 16 at 14:19










  • $begingroup$
    You would need to know that (eventually) $ln(n)<n^{0.5}$. There are proofs out there that (eventually) $ln(n)<n^{varepsilon}$ for any positive $varepsilon$.
    $endgroup$
    – alex.jordan
    Jan 16 at 15:41






  • 1




    $begingroup$
    For example, $ln(n)/n^{0.5}$. What is the limit of this as $ntoinfty$? By L'Hospital, it is the same as the limit of $1/(0.5n^{0.5})$, which is $0$. So for large enough $n$, $ln(n)/n^{0.5}<1$. So for large enough $n$, $ln(n)<n^{0.5}$.
    $endgroup$
    – alex.jordan
    Jan 16 at 16:39













Your Answer





StackExchange.ifUsing("editor", function () {
return StackExchange.using("mathjaxEditing", function () {
StackExchange.MarkdownEditor.creationCallbacks.add(function (editor, postfix) {
StackExchange.mathjaxEditing.prepareWmdForMathJax(editor, postfix, [["$", "$"], ["\\(","\\)"]]);
});
});
}, "mathjax-editing");

StackExchange.ready(function() {
var channelOptions = {
tags: "".split(" "),
id: "69"
};
initTagRenderer("".split(" "), "".split(" "), channelOptions);

StackExchange.using("externalEditor", function() {
// Have to fire editor after snippets, if snippets enabled
if (StackExchange.settings.snippets.snippetsEnabled) {
StackExchange.using("snippets", function() {
createEditor();
});
}
else {
createEditor();
}
});

function createEditor() {
StackExchange.prepareEditor({
heartbeatType: 'answer',
autoActivateHeartbeat: false,
convertImagesToLinks: true,
noModals: true,
showLowRepImageUploadWarning: true,
reputationToPostImages: 10,
bindNavPrevention: true,
postfix: "",
imageUploader: {
brandingHtml: "Powered by u003ca class="icon-imgur-white" href="https://imgur.com/"u003eu003c/au003e",
contentPolicyHtml: "User contributions licensed under u003ca href="https://creativecommons.org/licenses/by-sa/3.0/"u003ecc by-sa 3.0 with attribution requiredu003c/au003e u003ca href="https://stackoverflow.com/legal/content-policy"u003e(content policy)u003c/au003e",
allowUrls: true
},
noCode: true, onDemand: true,
discardSelector: ".discard-answer"
,immediatelyShowMarkdownHelp:true
});


}
});














draft saved

draft discarded


















StackExchange.ready(
function () {
StackExchange.openid.initPostLogin('.new-post-login', 'https%3a%2f%2fmath.stackexchange.com%2fquestions%2f3075320%2fdetermine-whether-the-series-sum-n-1-infty-frac-lnnn2-1-converg%23new-answer', 'question_page');
}
);

Post as a guest















Required, but never shown

























1 Answer
1






active

oldest

votes








1 Answer
1






active

oldest

votes









active

oldest

votes






active

oldest

votes









2












$begingroup$

Compare with $$sum_{n=1}^{infty}frac{1}{n^{1.5}}$$






share|cite|improve this answer









$endgroup$













  • $begingroup$
    you mean comparison not limit comparison?
    $endgroup$
    – hopefully
    Jan 16 at 4:56










  • $begingroup$
    Either way. This series converges. With direct comparison, the OP series has smaller terms (eventually). With a limit comparison, the ratio of the OP terms to the terms here converges to $0$. Either implies the OP series converges.
    $endgroup$
    – alex.jordan
    Jan 16 at 5:06










  • $begingroup$
    but if I use the direct comparison test how can I compare them?
    $endgroup$
    – hopefully
    Jan 16 at 14:19










  • $begingroup$
    You would need to know that (eventually) $ln(n)<n^{0.5}$. There are proofs out there that (eventually) $ln(n)<n^{varepsilon}$ for any positive $varepsilon$.
    $endgroup$
    – alex.jordan
    Jan 16 at 15:41






  • 1




    $begingroup$
    For example, $ln(n)/n^{0.5}$. What is the limit of this as $ntoinfty$? By L'Hospital, it is the same as the limit of $1/(0.5n^{0.5})$, which is $0$. So for large enough $n$, $ln(n)/n^{0.5}<1$. So for large enough $n$, $ln(n)<n^{0.5}$.
    $endgroup$
    – alex.jordan
    Jan 16 at 16:39


















2












$begingroup$

Compare with $$sum_{n=1}^{infty}frac{1}{n^{1.5}}$$






share|cite|improve this answer









$endgroup$













  • $begingroup$
    you mean comparison not limit comparison?
    $endgroup$
    – hopefully
    Jan 16 at 4:56










  • $begingroup$
    Either way. This series converges. With direct comparison, the OP series has smaller terms (eventually). With a limit comparison, the ratio of the OP terms to the terms here converges to $0$. Either implies the OP series converges.
    $endgroup$
    – alex.jordan
    Jan 16 at 5:06










  • $begingroup$
    but if I use the direct comparison test how can I compare them?
    $endgroup$
    – hopefully
    Jan 16 at 14:19










  • $begingroup$
    You would need to know that (eventually) $ln(n)<n^{0.5}$. There are proofs out there that (eventually) $ln(n)<n^{varepsilon}$ for any positive $varepsilon$.
    $endgroup$
    – alex.jordan
    Jan 16 at 15:41






  • 1




    $begingroup$
    For example, $ln(n)/n^{0.5}$. What is the limit of this as $ntoinfty$? By L'Hospital, it is the same as the limit of $1/(0.5n^{0.5})$, which is $0$. So for large enough $n$, $ln(n)/n^{0.5}<1$. So for large enough $n$, $ln(n)<n^{0.5}$.
    $endgroup$
    – alex.jordan
    Jan 16 at 16:39
















2












2








2





$begingroup$

Compare with $$sum_{n=1}^{infty}frac{1}{n^{1.5}}$$






share|cite|improve this answer









$endgroup$



Compare with $$sum_{n=1}^{infty}frac{1}{n^{1.5}}$$







share|cite|improve this answer












share|cite|improve this answer



share|cite|improve this answer










answered Jan 16 at 4:54









alex.jordanalex.jordan

39.3k560121




39.3k560121












  • $begingroup$
    you mean comparison not limit comparison?
    $endgroup$
    – hopefully
    Jan 16 at 4:56










  • $begingroup$
    Either way. This series converges. With direct comparison, the OP series has smaller terms (eventually). With a limit comparison, the ratio of the OP terms to the terms here converges to $0$. Either implies the OP series converges.
    $endgroup$
    – alex.jordan
    Jan 16 at 5:06










  • $begingroup$
    but if I use the direct comparison test how can I compare them?
    $endgroup$
    – hopefully
    Jan 16 at 14:19










  • $begingroup$
    You would need to know that (eventually) $ln(n)<n^{0.5}$. There are proofs out there that (eventually) $ln(n)<n^{varepsilon}$ for any positive $varepsilon$.
    $endgroup$
    – alex.jordan
    Jan 16 at 15:41






  • 1




    $begingroup$
    For example, $ln(n)/n^{0.5}$. What is the limit of this as $ntoinfty$? By L'Hospital, it is the same as the limit of $1/(0.5n^{0.5})$, which is $0$. So for large enough $n$, $ln(n)/n^{0.5}<1$. So for large enough $n$, $ln(n)<n^{0.5}$.
    $endgroup$
    – alex.jordan
    Jan 16 at 16:39




















  • $begingroup$
    you mean comparison not limit comparison?
    $endgroup$
    – hopefully
    Jan 16 at 4:56










  • $begingroup$
    Either way. This series converges. With direct comparison, the OP series has smaller terms (eventually). With a limit comparison, the ratio of the OP terms to the terms here converges to $0$. Either implies the OP series converges.
    $endgroup$
    – alex.jordan
    Jan 16 at 5:06










  • $begingroup$
    but if I use the direct comparison test how can I compare them?
    $endgroup$
    – hopefully
    Jan 16 at 14:19










  • $begingroup$
    You would need to know that (eventually) $ln(n)<n^{0.5}$. There are proofs out there that (eventually) $ln(n)<n^{varepsilon}$ for any positive $varepsilon$.
    $endgroup$
    – alex.jordan
    Jan 16 at 15:41






  • 1




    $begingroup$
    For example, $ln(n)/n^{0.5}$. What is the limit of this as $ntoinfty$? By L'Hospital, it is the same as the limit of $1/(0.5n^{0.5})$, which is $0$. So for large enough $n$, $ln(n)/n^{0.5}<1$. So for large enough $n$, $ln(n)<n^{0.5}$.
    $endgroup$
    – alex.jordan
    Jan 16 at 16:39


















$begingroup$
you mean comparison not limit comparison?
$endgroup$
– hopefully
Jan 16 at 4:56




$begingroup$
you mean comparison not limit comparison?
$endgroup$
– hopefully
Jan 16 at 4:56












$begingroup$
Either way. This series converges. With direct comparison, the OP series has smaller terms (eventually). With a limit comparison, the ratio of the OP terms to the terms here converges to $0$. Either implies the OP series converges.
$endgroup$
– alex.jordan
Jan 16 at 5:06




$begingroup$
Either way. This series converges. With direct comparison, the OP series has smaller terms (eventually). With a limit comparison, the ratio of the OP terms to the terms here converges to $0$. Either implies the OP series converges.
$endgroup$
– alex.jordan
Jan 16 at 5:06












$begingroup$
but if I use the direct comparison test how can I compare them?
$endgroup$
– hopefully
Jan 16 at 14:19




$begingroup$
but if I use the direct comparison test how can I compare them?
$endgroup$
– hopefully
Jan 16 at 14:19












$begingroup$
You would need to know that (eventually) $ln(n)<n^{0.5}$. There are proofs out there that (eventually) $ln(n)<n^{varepsilon}$ for any positive $varepsilon$.
$endgroup$
– alex.jordan
Jan 16 at 15:41




$begingroup$
You would need to know that (eventually) $ln(n)<n^{0.5}$. There are proofs out there that (eventually) $ln(n)<n^{varepsilon}$ for any positive $varepsilon$.
$endgroup$
– alex.jordan
Jan 16 at 15:41




1




1




$begingroup$
For example, $ln(n)/n^{0.5}$. What is the limit of this as $ntoinfty$? By L'Hospital, it is the same as the limit of $1/(0.5n^{0.5})$, which is $0$. So for large enough $n$, $ln(n)/n^{0.5}<1$. So for large enough $n$, $ln(n)<n^{0.5}$.
$endgroup$
– alex.jordan
Jan 16 at 16:39






$begingroup$
For example, $ln(n)/n^{0.5}$. What is the limit of this as $ntoinfty$? By L'Hospital, it is the same as the limit of $1/(0.5n^{0.5})$, which is $0$. So for large enough $n$, $ln(n)/n^{0.5}<1$. So for large enough $n$, $ln(n)<n^{0.5}$.
$endgroup$
– alex.jordan
Jan 16 at 16:39




















draft saved

draft discarded




















































Thanks for contributing an answer to Mathematics Stack Exchange!


  • Please be sure to answer the question. Provide details and share your research!

But avoid



  • Asking for help, clarification, or responding to other answers.

  • Making statements based on opinion; back them up with references or personal experience.


Use MathJax to format equations. MathJax reference.


To learn more, see our tips on writing great answers.




draft saved


draft discarded














StackExchange.ready(
function () {
StackExchange.openid.initPostLogin('.new-post-login', 'https%3a%2f%2fmath.stackexchange.com%2fquestions%2f3075320%2fdetermine-whether-the-series-sum-n-1-infty-frac-lnnn2-1-converg%23new-answer', 'question_page');
}
);

Post as a guest















Required, but never shown





















































Required, but never shown














Required, but never shown












Required, but never shown







Required, but never shown

































Required, but never shown














Required, but never shown












Required, but never shown







Required, but never shown







Popular posts from this blog

Can a sorcerer learn a 5th-level spell early by creating spell slots using the Font of Magic feature?

ts Property 'filter' does not exist on type '{}'

mat-slide-toggle shouldn't change it's state when I click cancel in confirmation window